Finish solving the system of equations.

x – 5y = 6

–x + 2y = –3

–3y = 3

What is the value of y?


Substitute the value of y back into one of the original equations to find the value of x. What is the value of x?

Answers

Answer 1

Answers:  x = 1 and y = -1

Explanation:

Start where your teacher left off, which is -3y = 3.

We solve for y by dividing both sides by -3

That turns -3y = 3 into y = -1

Then we use this y value to find x

x - 5y = 6

x - 5(-1) = 6

x + 5 = 6

x = 6-5

x = 1

The solution is (x,y) = (1, -1)

Answer 2

Answer:

-1,1

Step-by-step explanation:


Related Questions

Help me to find the product plz (opt math)

Answers

Answer:

hope it helps.stay safe healthy and happy...

Answer:

[tex]\left(sin\theta -cos\:a\right)\left(cos\:a+sin\theta \right)[/tex]

(sin(θ)-cos(a))(cos(a)+sin(0))

[tex]\mathrm{Apply\:Difference\:of\:Two\:Squares\:Formula:\:}\left(a-b\right)\left(a+b\right)=a^2-b^2[/tex]

a=sin(θ),b=cos(a)

= sin²(θ)-cos²(a)

-------------------------------

hope it helps...

have a great day!!

These points are linear.
Find the slope.
x-3 -2 -1 0 1/2
y-3 -2 -1 0 1 2
slope = [?]

Answers

Answer:

Slope = 4

Step-by-step explanation:

The slope of a linear equation can be calculated as;

m = y2-y1/x2-x1

What we have to do here is to select any two points ;

(x1,y1) = (3,8)

(x2,y2) = (6,20)

m = (20-8)/(6-3) = 12/3 = 4

A line with a slope of 3 passes through the point (-1, 2).

Write an equation for this line in point-slope form.

Answers

Answer:

Step-by-step explanation:

Slope = m = 3

(x₁ , y₁) = (-1 , 2)

Point slope form: y - y₁ = m(x - x₁)

y - 2 = 3(x - [-1] )

y - 2 = 3(x + 1)

y - 2 = 3*x + 3*1

y - 2 = 3x + 3

    y = 3x + 3 + 2

    y = 3x + 5

Quadrilateral A B C D is shown. The uppercase right angle, angle A, is 79 degrees.
What are the remaining angle measures if the figure is to be a parallelogram?

m∠B =
°

m∠C =
°

m∠D =
°

Answers

Answer:

m∠B =  

✔ 101

°

m∠C =  

✔ 79

°

m∠D =  

✔ 101

°

Step-by-step explanation:

Answer:

The answer above is right!

The correct answers are:

First box: option C. 101

Second box: option B. 79

Third box: option C. 101

Step-by-step explanation:

Just got it right on edge - Hope it helps :)

Brainliest would be greatly appreciated :D

a bag contains 3 white balls and 4 red balls. A.second bag contains 4 white balls and 3 red balls. If a bag is picked at random from each bag. Find the probability that both are white

Answers

Answer:

12/49 chance that they are both white

Step-by-step explanation:

We have 2 events going on:

The first bag that contains: 3 white, 4 red

The second bag that contains: 4 white, 3 red

First let's find the probability of picking a white ball from each individual bag (not combing the chances yet):

For the 1st bag, there are 3 white balls out of 7 balls (add 3 and 4).

That means there's a 3/7 chance you'll get a white ball from the 1st bag.

For the 2nd bag, there are 4 white balls out of 7 balls (add 4 and 3).

That means there's a 4/7 chance you'll get a white ball from the 2nd bag.

To find the probability of getting both white from both of the bags, multiply 3/7 & 4/7:

3/7 * 4/7 = 12/49

You can't simplify 12/49 further, so that's your answer!

Hope it helps (●'◡'●)

Answer:

2/6

Step-by-step explanation:

_______________________________hope it helps________

the answer is... 2/6

Can someone help me with this math homework please!

Answers

Step-by-step explanation:

these are almost the simplest possible problems to get you a little bit familiar with the use of this line of thinking and to use these methods.

come on ! are you really not understanding this ?

I think you are not trying to learn and get better, you try to get others doing your work for you.

these problems are so simple that stating them here costs you more time than just solving them.

remember fractions ?

remember the problems to find the smallest common denominator for a list of fractions ?

the first question is exactly that.

what is the smallest common denominator of 3/4, 1/3 and 1/2 ?

because that is the number you need to multiply both sides to eliminate all fractions.

so, what is the smallest number that can be split by 4, 3 and 2 ?

is it 2 ? no, 4 and 3 can't do that.

is it 3 ? no, 2 and 4 can't do that.

...

is it 6 ? no, 4 can't do that.

...

is it 8 ? no, 3 can't do that.

...

is it 12 ? yeah ! all 3 can do that.

here a little tip : you start by multiplying all involved numbers, and then work your way back by eliminating some factors shared by some of the numbers.

here it would be

2×3×4 = 24

but 12 is sufficient, because 2 and 4 share another common factor of 2. so, we can divide 24 by 2 and get 12.

"isolate the variable" means having it only on one side of the equation.

so, what do you think ?

subtracting any constants does not change anything for "j".

from all the offered options only the subtraction of 1.1j eliminates j from one side.

the runners save.

you only need to translate the words into the corresponding mathematical symbols.

runner 1 has $112 already saved.

that is our starting value for this runner.

so, 112 + ...

received a $45 gift card. so, $45 are added.

so, 112 + 45 + ...

saves $25 each month (m). so, whatever the number of m is, there will be $25×m added to the account.

so, 112 + 45 + 25m

we know already, it is the second option.

but for training purposes let's look at runner 2 too :

he starts with $50 already saved.

so, 50 + ...

saves $60 each month (m). same principle as above.

so, 50 + 60m

if we want to know, when (after how many months) both runners will have the same amount of money, we simply need to say both equations are equal and solve for m.

-3x + 1 + 10x = x + 4

how do we solve this ?

by bringing all x to one side and contract them into one expression. and try to bring all constants to the other side and contact them too. and then do the reverse operation of the x expression (like a division for a multiplication).

1 + 7x = x + 4

1 + 6x = 4

6x = 3

x = 3/6 = 1/2

hurray, it is the first option !

what do you understand by socialization​

Answers

Socialization is the lifelong process through which people learn the values and norms of a given society.

Determine the measure of <0

20.21°
0.005
73.74°
16.26°

Answers

Answer:

16.26°

Step-by-step explanation:

1. tanΘ= 7/24

2.[tex]tan^-1(\frac{7}{24} ) =[/tex]Θ

3. Θ = 16.26°

NEED HELPPPP RNNNNNNNN​

Answers

put it together first so
x+12-5x=3 solve from there it easy

CAN SOMEBODY PLEASE HELP MEEEE

Answers

Answer:

105.

Step-by-step explanation:

.

find the gradients of line a and b

Answers

Answer:

Gradient of A: 2

Gradient of B: -1

Step-by-step explanation:

Gradient = change in y/change in x

✔️Gradient of A using two points on line A, (2, 5) and (0, 1):

Gradient = (1 - 5)/(0 - 2) = -4/-2

Simplify

Gradient of A = 2

✔️Gradient of B using two points on line B, (0, 5) and (5, 0):

Gradient = (0 - 5)/(5 - 0) = -5/5

Simplify

Gradient of B = -1

How many black berry trees were sampled to create this graph

Answers

Answer:

C. 31 trees

Step-by-step explanation:

Add up all the frequencies: 3+3+8+10+5+2=31

Hope this helps!!!

Answer:

It's C. 31 trees

Step-by-step explanation:

That's about all I can say

Shirley buys fiction books for $20 each, and then marks up by 25% to
resell. What is the markup in dollars?

Answers

Answer:

$5

Step-by-step explanation:

Find the markup by finding 25% of 20:

20(0.25)

= 5

So, the markup is $5

Solve for the questions (both of them) and label you answers for which question

Answers

Jjbbh I don’t under stand

Which of the following recursive formulas represents the same arithmetic sequence as the explicit formula an = 5+ (n-1)2?

Answers

Answer:

[tex]a_1 = 5[/tex]

[tex]a_n = a_{n-1} + 2[/tex]

Step-by-step explanation:

Given

[tex]a_n = 5 + (n - 1)2[/tex]

Required

The equivalent recursive function

The general explicit function is:

[tex]a_n = a_1+ (n - 1)d[/tex]

So, by comparison

[tex]a_1 = 5[/tex]

[tex]d = 2[/tex]

The recursion of an arithmetic sequence is:

[tex]a_n = a_{n-1} + d[/tex]

Substitute 2 for d

[tex]a_n = a_{n-1} + 2[/tex]

Hence: (a) is correct

If anyone can help with this ill mark Brainly

Answers

Answer:

In this case, we can do substitution.

Step-by-step explanation:

For the first one, (s - t)(x) = ((x - 5) - 4x^2)(x) = x^2 - 21x

For the second one, (s*t)(x) = ((x - 5) *4x^2)(x) = 4x^4 - 20x^3

And for the last one,  (s+t)(-2) = ((x - 5) + 4x^2)(-2) = -8x^2 - 2x + 10

Hope your happy with the answer :)

Hello, Brainly community!

This question is for all of those Calculus people out there.

The volume of a swimming pool is changing with respect to time, such that the volume is given by W(t), where W(t) is measured in cubic centimeters and t is measured in seconds. A tangent line is shown for W(t) at t = 3 seconds. Determine the best estimate for the value of the instantaneous rate of change of W(t) when t = 3.
(I've narrowed down the answer choices to 2, and just really need to find the right way of thinking to find the answer)

(A) W(lim t) as t goes to 3.
(B) [W(3.1) - W(2.9)] / 0.2.

Thank you in advance!

Answers

Answer:

(B)  [tex]\displaystyle \frac{W(3.1) - W(2.9)}{0.2}[/tex]

General Formulas and Concepts:

Calculus

Limits

Derivatives

The definition of a derivative is the slope of the tangent line.

Derivative Notation

Instantaneous Rates

Tangent Line: [tex]\displaystyle f'(x) = \frac{f(b) - f(a)}{b - a}[/tex]

Step-by-step explanation:

Since we are trying to find a rate at which W(t) changes, we must find the derivative at t = 3.

We are given 2 close answer choices that would have the same numerical answer but different meanings:

(A)  [tex]\displaystyle \lim_{t \to 3} W(t)[/tex](B)  [tex]\displaystyle \frac{W(3.1) - W(2.9)}{0.2}[/tex]

If we look at answer choice (A), we see that our units would simply just be volume. It would not have the units of a rate of change. Yes, it may be the closest numerically correct answer, but it does not tell us the rate at which the volume would be changing and it is not a derivative.

If we look at answer choice (B), we see that our units would be cm³/s, and that is most certainly a rate of change. Answer choice (B) is also a derivative at t = 3, and a derivative tells us what rate something is changing.

∴ Answer choice (B) will give us the best estimate for the value of the instantaneous rate of change of W(t) when t = 3.

Topic: AP Calculus AB/BC (Calculus I/I + II)

Unit: Differentiation

Book: College Calculus 10e

Drag


the yellow point until an accurate "height" of the triangle is drawn. Afterwards, fill out the empty boxes below to determine the area of the triangle.

Answers

Answer:

I don't see a yellow point

Which number completes the system of linear
inequalities represented by the graph?
y> 2x – 2 and x + 4y > _____

Answers

The system of linear inequalties that is represented by the graph is formed by the expressions y > 2 · x - 2 and x + 4 · y > - 12.

What system of inequalities generate the graph seen in the image?

Herein we have two inequalities generated by two linear functions whose form is described below:

f(x, y) > a (blue region)      (1)

g(x, y) > b (red region)      (2)

By direct comparison, we find that the limit of the red region is represented by the inequality y > 2 · x - 2. From the equation of the line we can derive an expression for the limit of the blue region:

Slope

m = [- 4 - (- 3)]/(4 - 0)

m = - 1/4

Intercept

b = y - m · x

b = - 3 - (- 1/4) · 0

b = - 3

Then, the inequation is represented by the linear equation:

y > - (1/4) · x - 3

4 · y > - x - 12

x + 4 · y > - 12

The system of linear inequalties that is represented by the graph is formed by the expressions y > 2 · x - 2 and x + 4 · y > - 12.

To learn more on inequalities: https://brainly.com/question/20383699

#SPJ1

Which is the area between the x-axis and y=x from x=1 to x=5

Answers

Answer:

[tex]\displaystyle A = 12[/tex]

General Formulas and Concepts:

Pre-Algebra

Order of Operations: BPEMDAS

BracketsParenthesisExponentsMultiplicationDivisionAdditionSubtractionLeft to Right

Algebra I

FunctionsFunction NotationGraphing

Calculus

Integrals

Definite IntegralsArea under the curve

Integration Rule [Reverse Power Rule]:                                                               [tex]\displaystyle \int {x^n} \, dx = \frac{x^{n + 1}}{n + 1} + C[/tex]

Integration Rule [Fundamental Theorem of Calculus 1]:                                     [tex]\displaystyle \int\limits^b_a {f(x)} \, dx = F(b) - F(a)[/tex]

Area of a Region Formula:                                                                                     [tex]\displaystyle A = \int\limits^b_a {[f(x) - g(x)]} \, dx[/tex]

Step-by-step explanation:

Step 1: Define

Identify

y = x

Interval: x = 1 to x = 5

Step 2: Sort

Graph the function. See Attachment.

Bounds of Integration: [1, 5]

Step 3: Find Area

Substitute in variables [Area of a Region Formula]:                                   [tex]\displaystyle A = \int\limits^5_1 {x} \, dx[/tex][Integral] Integrate [Integration Rule - Reverse Power Rule]:                     [tex]\displaystyle A = \frac{x^2}{2} \bigg| \limits^5_1[/tex]Evaluate [Integration Rule - Fundamental Theorem of Calculus 1]:           [tex]\displaystyle A = 12[/tex]

Topic: AP Calculus AB/BC (Calculus I/I + II)

Unit: Integration

Book: College Calculus 10e

Identify the parts of the following algebraic expression.

-8z + 1
2
y - 7.7

Term:
Variable:
Coefficient:
Constant:

Answers

Answer:

-8z+1

term:2

variable:z

coefficient:-8

constant:1

2

term:1

variable:nil

coefficient:nil

constant:2

y-7.7

term:2

variable:y

coefficient:nil

constant:-7.7

The solution is given below.

What is number?

A number is a mathematical object used to count, measure, and label. The original examples are the natural numbers 1, 2, 3, 4, and so forth. Numbers can be represented in language with number words.

now, we get,

-8z+1

term:2

variable: z

coefficient:-8

constant:1

again,

2

term:1

variable : nil

coefficient : nil

constant:2

now,

y-7.7

term:2

variable : y

coefficient : nil

constant:-7.7

To learn more on number click:

brainly.com/question/17429689

#SPJ2

Question: Dentify the parts of the following algebraic expression.

-8z + 1

2

y - 7.7

Term:

Variable:

Coefficient:

Constant:

PLZ HELP its due soon!!!

Answers

Answer:

E)

Step-by-step explanation:

1) 6.50 (starting with his current wage)

2) 6.50 + 0.25 = 6.75 $

3) 6.75 + 0.25 = 7.00 $

4) 7.00 + 0.25 = 7.25 $

5) 7.25 + 0.25 = 7.50 $

6) 7.50 + 0.25 = 7.75 $

Can someone help with problems fivethroughseven

Answers

Answer:

5) 15120

6) 11880

7) 336

Step-by-step explanation:

The formula for permutation where mPn is m!/(m-n)!

Applying this to question 5, we get 9!/4!, which is 15120.

For question 6, we get 12!/8!, which is 11880.

For question 7, we get 8!/5!, which is 336.

Which of the following is not a way to represent the solution of the inequality 2(x − 1) greater than or equal to −12? (1 point)

x greater than or equal to −5
x less than or equal to −5
−5 less than or equal to x
A number line with a closed circle on negative 5 and shading to the right

Answers

Answer:

x less than or equal to -5

Step-by-step explanation:

2(x-1) ≥ -12

2x - 2 ≥ -12

2x ≥ -10

x ≥ -5

Graph the solution of the inequality 3/7(35x-14)<_ 21x/2+3

Answers

Answer:

You'll have a closed circle at x = 2, and shading to the left

See the diagram below

=========================================================

Explanation:

The fractions here are 3/7 and 21/2. The denominators of which are 7 and 2 respectively. The LCD is 7*2 = 14.

If we multiply both sides by 14, then this will clear out the denominators and make the fractions go away.

14*(3/7) = (14*3)/7 = 42/7 = 814*(21/2) = (14*21)/2 = 294/2 = 147

So if we multiplied both sides by 14, then we have these steps

[tex]\frac{3}{7}(35x-14) \le \frac{21x}{2}+3\\\\14*\frac{3}{7}(35x-14) \le 14*\left(\frac{21x}{2}+3\right)\\\\14*\frac{3}{7}(35x-14) \le 14*\left(\frac{21x}{2}\right)+14*\left(3\right)\\\\6(35x-14) \le 147x+42\\\\[/tex]

--------------------------

Let's isolate x

[tex]6(35x-14) \le 147x+42\\\\6(35x)+6(-14) \le 147x+42\\\\210x-84 \le 147x+42\\\\210x-147x \le 42+84\\\\63x \le 126\\\\x \le 126/63\\\\x \le 2\\\\[/tex]

The graph of this will consist of a closed or filled in circle at x = 2. We shade to the left to represent numbers smaller than 2.

So either x = 2 or x < 2.

If we used an open hole at 2, then we wouldn't be including 2 (but we want to include this endpoint).

See the diagram below.

**Does anybody knows how to do it***

instructions: state what additional information is required in order to know that the triangle are congruent for the given reason.

Given: ASA

Answers

With ASA, we need two angles and an included side. An included side is one that is between the two angles.

But, we need to make sure the sides match up. So, triangle KJI is congruent to triangle ZYX.

Therefore, we need to know that side JI is congruent to YX.

Hope this helps!

What is the area of a cross section that is parallel to face BFGC?

Answers

Answer: 216cm2

Step-by-step explanation: 36x6

URGENT PLS ANSWER QUICKLY​

Answers

Answer:

9th is 44 mark me as brainlist

Answer:

Question 9 is perimeter

Help!!!
Find the domain of the function. Write the answer in interval notation.

Answers

Answer:

A

Step-by-step explanation:

Given

f(x) = [tex]\frac{2}{3x+2}[/tex]

The denominator cannot be zero as this would make f(x) undefined.

Equating the denominator to zero and solving gives the value that x cannot be.

3x + 2 = 0 ⇒ 3x = - 2 ⇒ x = - [tex]\frac{2}{3}[/tex] ← excluded value

Then

domain is ( - ∞ , - [tex]\frac{2}{3}[/tex] ) U ( - [tex]\frac{2}{3}[/tex], ∞ ) → A

There is $1.90 in a jar filled with
quarters, dimes, and nickels.
There are 2 more quarters than
dimes and there are 2 more
nickels than quarters.
How many of each coin are there?

Answers

Answer:

7 nickels, 5 quarters, 3 dimes

Step-by-step explanation:

7 nickels= 35 cents

5 quarters= $1.25

3 dimes= 30 cents

35+ 1.25+ 30= $1.90

Hope this helps!

Plz mark Brainliest if u can :)

Other Questions
A bottle maker believes that 23% of his bottles are defective. If the bottle maker is accurate, what is the probability that the proportion of defective bottles in a sample of 602 bottles would differ from the population proportion by less than 4% A piece of wood is cut into three pieces in the ratio 6: 5: 2. If the log is 61/2 feet long, what will be the length of the longest piece my friend said,"where are they staying" change into indirect speech. Calculate the length of the side AB, knowing that AC = 2, m (C) = 30 and BC = 4. Help ..! I really need help with this problem Archie took several deep breaths of air. By breathing deeply, what mechanism did he assist to return venous blood to his heart The following two waves are sent in opposite directions on a horizontal string so as to create a standing wave in a vertical plane: y1(x, t) = (8.20 mm) sin(4.00x - 430t) y2(x, t) = (8.20 mm) sin(4.00x + 430t), with x in meters and t in seconds. An antinode is located at point A. In the time interval that point takes to move from maximum upward displacement to maximum downward displacement, how far does each wave move along the string? D State Six Impact of ICT the society Solve the inequality and express your answer in interval notation x^2 - 12x + 3 6. Two cell phone companies charge a flat fee plus an added cost for each minute or part of a minute used. The cost is represented by C and the number of minutes is represented by t.Call-More: C = 0.40t + 25 Talk-Now: C = 0.15t + 40a) Which company is cheaper if a customer talks for 50 minutes. (1 mark)b) Under what conditions do the two companies charge the same? (3 marks)c) Under what conditions is Talk-Now better? Explain could someone help me with this ASAP? The physiologic effects of ______ include blood vessel constriction and decreased cellular metabolism. An example of an established sociological theory is:Group of answer choicesfactor analysissymbolic interactionismsocial realismall of the above What is the value of x in the figure below? If necessary, round your answer tothe nearest tenth of a unit.X15D 4 BA. 7.7B. 3.8O C. 15D. 4 I dont think I got the right answer? The graph of f(x)=x^2 is shown. Compare the graph of f(x) with the graph of d(x)=x^2-26 1. Show that heat flows spontaneously from high temperature to low temperature in any isolated system (hint: use entropy change that occurs during the process for your proof).2. Work out the entropy change for the decomposition of mercuric oxide using mathematical and graphical arguments. How many fixed-requirement constraints does a transportation problem with 5 factories and 6 customers have?a. 30.b. 11.c. 6.d. 5. Imagine you ordered a printer online two weeks ago. You paid for delivery within 24 hours but it hasnt arrived. Write an email to the supplier and request a refund. ? can you invision yourself being involved in the arts in some way PLZZ HELPPPPPP I NEED TO FINISH THIS IN 8 MINS